please help meeee i am lost

Please Help Meeee I Am Lost

Answers

Answer 1

Answer:
The sum of a sequence is known as a series. Thus means a harmonic series is infinite with no limit


Related Questions

Hannah's friend Ami would like the group of 5 performers to include more.
males than females. The order in which they perform is no longer
relevant.
iii) Find the number of different selections of 5 performers with more
males than females.
iv) Two of the applicants areMr andMrs Blake. Find the number of
different selections that include Mr and Mrs Blake and also fulfil Ami's.
requirement.

Answers

Answer:

iii) 501

iv) 110

Step-by-step explanation:

See picture

The population of an endangered species of fish living in a controlled habitat is given by the equation P = 7t + 46, where Pis the population and t is the time in months since the population was introduced into the habitat. Graph the equation using t = 0, 6, 12, and 18. (100 pts)

Answers

Graph is attached. It will be a linearly increasing line.

Population for 0 months is 46

Population for 6 months is 88

Population for 12 months is 130

Population for 18 months is 172.

Given,

The equation for population, P = 7t + 46

P is the population

t is the time in months

We have to graph the equation using

t = 0, 6, 12 and 18.

For this we have to find P

1. t = 0

P = 7t + 46 = 7 × 0 + 46 = 0 + 46 = 46

2. t = 6

P = 7t + 46 = 7 × 6 + 46 = 42 + 46 = 88

3. t = 12

P = 7t + 46 = 7 × 12 + 46 = 84 + 46 = 130

4. t = 18

P = 7t + 46 = 7 × 18 + 46 = 126 + 46 = 172

The graph is given below and it will be a linearly increasing line.

Learn more about line graph here;

https://brainly.com/question/23680294

#SPJ1

Estimate.45.86 + 54.14A.90B.99C.100D.110

Answers

We need to estimate the sum:

[tex]45.86+54.14[/tex]

In order to do so, we first round each term to the nearest integer. Then we add them.

We have:

[tex]\begin{gathered} 45.86\cong46\text{ (because 0.86>0.50)} \\ \\ 54.14\cong54\text{ (because 0.14<0.50)} \end{gathered}[/tex]

Then, we can estimate the sum to be:

[tex]45.86+54.14\cong46+54=100[/tex]

Therefore, option C is correct.

Isotope: 239Pu
Half-life
(years): 24,100
Initial
Quantity:
Amount After
4000 Years:
3.7g

Answers

The mass of isotope 239-Pu left after 4000 years is approximately 3.3g

Radioactivity

Radioactivity is the phenomenon of the spontaneous disintegration of unstable atomic nuclei to atomic nuclei to form more energetically stable atomic nuclei. Radioactive decay is a highly exoergic, statistically random, first-order process that occurs with a small amount of mass being converted to energy.

In solving the disintegration constant, we can use the half-life formula.

The half-life of a sample is given as;

[tex]T_\frac{1}{2} = \frac{ln2}{\lambda}[/tex]

Substituting the values into the equation;

[tex]T_\frac{1}{2} = \frac{ln2}{\lambda}\\24100 = \frac{ln2}{\lambda} \\\lambda = \frac{ln2}{24100} \\\lambda = 0.00002876years^-^1[/tex]

The formula of radioactivity is given as

[tex]N = N_oe^(^-^\lambda ^t)[/tex]

No = 3.7gt = 4000

We can plug in the values and solve.

[tex]N = N_oe^(^-^\lambda ^t)\\N = 3.7e^-^(^0^.^0^0^0^0^2^8^7^6 ^*^4^0^0^0^)\\N = 3.297g\\N = 3.3g[/tex]

The amount left after 4000 years is 3.3g.

Learn more on radioactivity here;

https://brainly.com/question/26626062

#SPJ1

HELP ME PLEASE LIKE PLEASE I DONT UNDERSTAND THIS MATH

Answers

Given that:

- A race car game takes 6 points each time the player hits a cone.

- You must find the integer that represents the change in total points if the player hits 10 cones.

It is important to remember that the Set of Integers contains the negative numbers, the positive numbers, and zero.

Then, by analyzing the data given in the exercise, you can identify two integers that must be multiplied, in order to calculate the total points if the player hits 10 cones. These are:

[tex]\begin{gathered} 10\text{ } \\ -6 \end{gathered}[/tex]

Since the game takes 6 points from the player each time this hits a cone, the integer that represents the points taken from the player must be negative.

In order to multiply the integers, you need to remember the Sign Rules for Multiplication:

[tex]\begin{gathered} +\cdot+=+ \\ -\cdot-=+ \\ +\cdot-=- \\ -\cdot+=- \end{gathered}[/tex]

Therefore, you get this result:

[tex](10)(-6)=-60[/tex]

Hence, the answer is:

For the square ABCD, it is known that | AB| = | AC| and |DA| = |DC| and corner ABC =
corner CDA. One of the corners of this square is 150 degrees. Find the sizes of the three remaining angles of the square ABCD.

Answers

Answer:

∠BAD = 150°∠ABC = 48°∠BCD = 114°∠ADC = 48°

Step-by-step explanation:

Given quadrilateral ABCD with AB≅AC, DA≅DC, ∠ABC≅∠ADC, and one of the corner angles equal to 150°, you want the measures of all corner angles.

Setup

The given congruent angles and sides are marked in the attached diagram. If we let ∠ACB = x and ∠ACD = y, we can write some equations involving the sums of the various angles.

First of all, we note that both triangles are isosceles. That means ...

∠ABC≅∠ACB = x∠DAC≅∠DCA = y

The sum of angles in ∆ACD must be 180°, so we have ...

  x + 2y = 180°

We know that the isosceles triangle base angles cannot exceed 90°, so the angles at corners B and D cannot be 150°. That leaves two possibilities:

(a) Corner angle C is 150°   ⇒   x + y = 150°

(b) Corner angle A is 150°   ⇒   (180°-2x) +y = 150°

Solution

(a) Corner C is 150°

This makes the system of equations for x and y be ...

x +2y = 180°x +y = 150°

Subtracting the second from the first gives y = 30°, and substituting that value for y gives x = 120°. We already know x is the base angle of an isosceles triangle, so cannot have that value. This possibility is eliminated.

(b) Corner D is 150°

This makes the system of equations for x and y be ...

x +2y = 180°(180° -2x) +y = 150°

The second of these equations can be rearranged to ...

  2x -y = 30°

Adding twice this to the first equation, we have ...

  2(2x -y) +(x +2y) = 2(30°) +(180°)

  5x = 240°

  x = 48°

Substituting for x in the second equation gives ...

  2(48°) -y = 30°

  96° -30° = y = 66°

As we noted in the solution part (a), corner angle C is the sum of x and y:

  ∠BCD = x +y = 48° +66° = 114°

Corner angles

Then the corner angles are ...

∠BAD = 150°∠ABC = 48°∠BCD = 114°∠ADC = 48°

The attached figure is drawn to scale.

Give two real-world situations that could be represented by the value -2

Answers

We have to represent two real world situations that could be represented by the value -2.

(1) Let there be a fish which is 2 meter below the sea level.

Considering the sea level point to be zero and denoting the points above sea level as positive and the points below sea level as negative, like we do in a cartesian plane, the point which is 2 meter below the ground level will be -2.

(2) Let there be a box buried 2 feet under the ground.

Considering the ground level to be zero and denoting the point above the ground level as positive and the points below ground level as negative, the point which is 2 meter below the ground level will be -2.

To learn more about cartesian plane, here:-

https://brainly.com/question/28574364

#SPJ1

20 more than half a number is 70 what is the number

Answers

The expression for this problem can be shown as:

1/2n + 20 = 70

where n represents the number. We can solve by isolating the variable. To do this, we subtract 20 on both sides to make the equation become:

1/2n = 50

Let’s divide both sides by 1/2 to find that:

n = 100

We can substitute 100 into the initial equation for verification:

1/2(100) + 20 = 70

50 + 20 = 70

Since this is true, we now know that the number is 100.

Multiply.3/5⋅3/4Enter your answer as a fraction in simplest form in the box.

Answers

Hello!

We have the following multiplication:

[tex]\frac{3}{5}\times\frac{3}{4}[/tex]

To solve it, we have to multiply numerator with numerator and denominator with denominator, look:

[tex]\frac{3}{5}\times\frac{3}{4}=\frac{3\times3}{5\times4}=\frac{9}{20}[/tex]

So, the fraction in is the simplest form is 9/20.

I need help with this question but no one is helping me! Can someone please help me.​

Answers

Step-by-step explanation:

"with replacement" is important.

that means that the mixture of the marbles and therefore the chances to pull certain marbles does not change with the previous pull.

a probability is always

desired cases / totally possible cases

we have 14 marbles in the bag

4 orange

2 green

8 purple

the probability to pull an orange marble is therefore

4/14 = 2/7

the probability to pull a green marble is therefore

2/14 = 1/7

the probability to pull a purple marble is therefore

8/14 = 4/7

a.

a green marble is pulled AND then a purple marble is selected.

we need to combine both probabilities for one combined event :

1/7 × 4/7 = 4/49

b.

a purple marble and then a green marble is selected.

4/7 × 1/7 = 4/49

since we replace the marbles after every pull, both probabilities are the same, of course.

every pull has the same probability for each desired event.

c.

2 orange marbles are selected.

2/7 × 2/7 = 4/49

d.

2 non-orange marbles are pulled

there are 10 non-orange marbles. so this is

10/14 × 10/14 = 5/7 × 5/7 = 25/49

Given f(x)=-2x-1f(x)=−2x−1, solve for x when f(x)=-7f(x)=−7

Answers

The value of the function f(x) = 2x -1 when f(x) = 7 is 13.

Function:

A function is defined as a relation from a set of inputs to a set of possible outputs where each input is related to exactly one output.

Given,

Here we have the function f(x) = 2x - 1.

Now, we need to find the value of the function when f(7).

To find the value of f(7), we have to replace the value of x with the value 7.

Then we get the equation like the following,

f(7) = 2(7) - 1

Now, we have to expand the terms in order to solve it,

f(7) = 14 - 1

When we subtract it, then we get the value of

f(7) = 13.

Therefore, the value of f(7) = 13.

To know more about Function here.

https://brainly.com/question/12431044

#SPJ1

A random sample of 269 student were asked what kind of vehicle they prefer a car truck the following contingency table gives the 2 way classification of the responses

Answers

Given a random sample of 269 students and a contigency table that gives the 2 way classification of their responses.

The probability formula is:

[tex]P(E)=\frac{n(\text{Required outcome)}}{n(\text{Possible outcome)}}[/tex][tex]P(\text{Female)}=\frac{(99+22)}{269}=\frac{121}{269}=0.450[/tex][tex]P(\text{Car)}=\frac{(87+99)}{269}=\frac{186}{269}=0.691[/tex][tex]\begin{gathered} P(\text{Female}|\text{Truck)}=\frac{P(\text{Female }\cap\text{ Truck)}}{P(\text{Truck)}} \\ =\frac{22}{(61+22)}=\frac{22}{83}=0.265 \end{gathered}[/tex][tex]P(\text{Truck }\cap\text{ Female)=}\frac{22}{(99+22)}=\frac{22}{121}=0.182[/tex]

The conditional probability P(A/B) or P(B/A) arises only in the case of dependent events.

The graph of F(x), shown below, resembles the graph of G(x) = x², but it hasbeen changed somewhat. Which of the following could be the equation ofF(x)?OA Fx)=0.3x²+2 OB. F(x)=3x2+2OC. F(x) = -x²2² +2D. F(x)=x²+2

Answers

Answer:

Explanation:

Given the function:

[tex]G(x)=x^2[/tex]

The graph of G(x) is wider than the graph of F(x), so we can conclude that G(x) has been stretched vertically by a factor greater than 1 to obtain F(x).

Furthermore, G(x) was translated upwards by 2 units when we compare the vertices.

Thus, from the options,a possible equation for F(x) is:

[tex][/tex]

Answer:F(x)=3x^2+2

Step-by-step explanation:

Write the system of equations associated with the given augmented matrix. Do not solve

Answers

The elements in the first 3 columns are the coefficients of system of equations.

The 4th column are the constants of the system of equations.

This is a system of 3 equations.

Let's write the system using the variables x, y, and z. Shown below:

[tex]\begin{gathered} 1x+1y+0z=3 \\ 0x+4y+1z=-5 \\ 1x+0y-1z=4 \end{gathered}[/tex]

Let's simplify and write the equations:

[tex]\begin{gathered} x+y=3 \\ 4y+z=-5 \\ x-z=4 \end{gathered}[/tex]

i need help can anyone help me please??

Answers

Hence the second option, [tex]y=\frac{2}{3}x +\frac{11}{3}[/tex] is the correct option.

What is slope interception form?

Y=mx+b, where m is the slope and b is the y-intercept, is the slope intercept form. The graph of the linear equation can be drawn on the x-y coordinate plane using this form of the equation.

For the given question we are given,

slope (m) = 2/3

and we are also given the point that the line passes through to be,

(-1,3).

that is

(x,y)=(-1,3)

x=-1

y=3

we, know that slope interception form is of the form,

y=mx+c

substituting the value of m (slope) and the point (-1,3) we get

3=(2/3)*(-1)+b

=   3= -2/3 +b

=   3+2/3=b

=    (9+2)/3=b

=     11/3=b

from this we get b=11/3

now substituting this again in

y=mx+b

we get

y=(2/3)x+(11/3)

Hence the second option [tex]y=\frac{2}{3}x +\frac{11}{3}[/tex] is correct.

to learn more about slope interception form visit:

https://brainly.com/question/1884491

#SPJ9

You have $12 left to spend. Here are your options: $12 for 6 goodie bags for your guests or $12 for 4 goodie bags for your guests. We want to get the most for what our money can buy. Show your work in the space below. Which is the better deal?

Answers

For the first deal we get 6 goodie bags for $12, let's find how much each goodie bag cost in this case by dividing $12 by 6:

[tex]\frac{12}{6}=2[/tex]

In the first deal, each goodie bag costs $2.

For the second deal, we get 4 goodie bags for $12, let's also find how much is 1 goodie bag in the scenario:

[tex]\frac{12}{4}=3[/tex]

In the second deal, each goodie bag costs $3.

The better deal is the first one because each goodie bag is cheaper than in the second deal.

Answer: $12 for 6 goodie bags is the better deal

at 3:00 am the temperature was -8 degrees.By early evening, the temperature rose 25 degrees at late day the temperature dropped 5 degrees. What was the late day temperature?

Answers

According to the problem:

• At 3:00 am the temperature was -8°.

,

• Then, it rose 25 degrees. ,This implies a sum.

,

• At last, it dropped 5 degrees. ,This implies a subtraction.

Remember that "rising" shows a positive direction and "dropping" shows a negative direction. Following this rule, we can form the following expression

[tex]T=-8+25-5[/tex]

There you can observe that the initial temperature (-8) increased (+)25, and then decreased (-)5.

Therefore, the temperature for the late day is

[tex]T=12[/tex]

372.2 is what percent of 642

Answers

Answer:

57.97507788%

Step-by-step explanation:

Write the problem as a mathematical expression.

372.2

642    

Multiply by 100 to convert to a percentage.

372.2

642     * 100

Simplify

372.2.

642.      * 100 = 57.97507788%

calculate the measure of SR

Answers

In the given figure

There is a triangle RST

∵ RU is perpendicular on ST and bisects it

∴ Triangle RST is an isosceles triangle

∴ RS = RT

∵ RS = 3x + 9 and RT = 7x + 17

Equate them

∵ 7x + 17 = 3x + 9

Subtract 3x from both sides

∴ 7x - 3x + 17 = 3x - 3x + 9

∴ 4x + 17 = 9

Subtract 17 from both sides

∵ 4x + 17 - 17 = 9 - 17

∴ 4x = -8

Divide both sides by 4 to find x

∴ x = -2

Now substitute x by -2 in the expression of RS to find its length

∵ RS = 3(-2) + 9

∴ RS = -6 + 9

RS = 3

These two trangles are scaled copies of one another. the area of the smaller triangle is 9 square
units.

Answers

The area of the larger triangle will be equal to 36 square units.

Triangle may be defined as a closed figure of three sides and three interior angles. Scaling of image may be defined as a ratio that can be used to represent the relationship between the shape and size of a figure and the corresponding dimensions of the actual figure or object. Dilation may be defined as the process for creating similar figures by changing the size. A graph can be defined as a pictorial representation or a diagram that represents data or values. According to question the area of smaller triangle is given as 9 square units. After scaling the length of scale copies become double and the area becomes four times. So, the area of bigger triangle is 4×9 = 36.

Learn more about the Scale images here:

brainly.com/question/13194929

#SPJ9

Complete Question:

These two triangles are scaled copies of one another. The area of the smaller triangle is 9 square units. What is the area of the larger triangle? Explain or show how you know.

Every hour on the hour, Helen does jumping jacks. At 4:00 she does 4 jumping jacks. At 5:00 she does 5 jumping jacks, etc. How many jumping jacks will Helen do, in total, from the time prior to her final jumping jacks at 4:00 to the time prior to her first jumping jacks at 11:00 ?a) 46b) 56c) 57d) 60

Answers

We have to add the jumping jacks from 4:00 (not included) to 11:00.

So we have to add:

[tex]\sum ^{11}_{n\mathop=5}n=5+6+7+8+9+10+11=56[/tex]

The answer is option b) 56.

Chewbacca is standing from atop a plateau 500 feet in the air looking down ata Jedi Trainee. If the Jedi Trainee is 150 feet away from the base of theplateau, what is the angle of depression from Chewbacca to the Jedi Trainee ifChewbacca's eye height is 7.25 feet?Show your work here and explain how you arrive at your answer.

Answers

In the given situation the angle of depression is θ. To find it, use the trigonometric ratio of tangent of angle θ in a right triangle (the ratio of tangent is the relation between the two legs of a right traingle):

[tex]\tan \theta=\frac{opposite\text{ leg}}{adjacent\text{ leg}}[/tex]

In the given situation the opposite leg to the angle of depresion is the distance between the base of the plateau and the Jedi Trainee and the adjacent leg is the sum of 500ft in the air and Chewbacca's eye height:

[tex]\begin{gathered} \tan \theta=\frac{150ft}{500ft+7.25ft} \\ \\ \tan \theta=\frac{150}{507.25} \\ \\ \tan ^{-1}(\tan \theta)=\tan ^{-1}(\frac{150}{507.25}) \\ \\ \theta=\tan ^{-1}(\frac{150}{507.25}) \\ \\ \theta\approx16.47 \end{gathered}[/tex]Then, the angle of depression from Chewbacca to the Jedi Trainee is approximately 16.47°

Which equation in standard form has a graph that passes through the point (2,19) and has a slope of 11/2

Answers

Answer:

y=5.5x+8

Step-by-step explanation:

Replace x and y values so you get 19=5.5(2)+cMultiply 5.5 by 2 to get 19=11+cSubtract 11 from both sides to get 8=cSimplify to y=5.5x+8

If the first two terms of a fibonacci sequence are 5, 25 then what us the next term ?

Answers

30,60,90,

Step-by-step explanation:

Find the solution set to each inequality 6m + 2 < 5m - 4-3(x-7) > -278(p-6) - 4(p-4)

Answers

The inequality to solve is,

[tex]8(p-6)>4(p-4)[/tex]

We will use the distributive property (shown below) to simplify the expression. Then, we will use algebra rules to solve the inequality.

Distributive Property

[tex]a(b-c)=ab-ac[/tex]

The simplification process >>>

[tex]\begin{gathered} 8(p-6)>4(p-4) \\ 8(p)-8(6)>4(p)-4(4) \\ 8p-48>4p-16 \\ 8p-4p>-16+48 \\ 4p>32 \\ \frac{4p}{4}>\frac{32}{4} \\ p>8 \end{gathered}[/tex]Answerp > 8

If cos(theta) = 5/0 and is in the 1st quadrant, find the following:

Answers

step 1

Find out sine

Remember the identity

[tex]cos^2\theta+sin^2\theta=1[/tex]

substitute the given value of cosine

[tex]\begin{gathered} (\frac{5}{9})^2+s\imaginaryI n^2\theta=1 \\ \\ s\imaginaryI n^2\theta=1-\frac{25}{81} \\ \\ s\mathrm{i}n^2\theta=\frac{56}{81} \\ \\ sin\theta=\frac{\sqrt{56}}{9} \\ \\ sin\theta=\frac{2\sqrt{14}}{9} \end{gathered}[/tex]

step 2

Find out cosecant

[tex]\begin{gathered} csc\theta=\frac{1}{sin\theta} \\ \\ csc\theta=\frac{9}{2\sqrt{14}}*\frac{\sqrt{14}}{\sqrt{14}}=\frac{9\sqrt{14}}{28} \\ \\ csc\theta=\frac{9\sqrt{14}}{28} \end{gathered}[/tex]

step 3

Find out secant

[tex]\begin{gathered} sec\theta=\frac{1}{cos\theta} \\ \\ sec\theta=\frac{9}{5} \end{gathered}[/tex]

step 4

Find out tangent

[tex]\begin{gathered} tan\theta=\frac{sin\theta}{cos\theta} \\ \\ tan\theta=\frac{\frac{2\sqrt{14}}{9}}{\frac{5}{9}}=\frac{2\sqrt{14}}{5} \end{gathered}[/tex]

step 5

Find out cotangent

[tex]\begin{gathered} cot\theta=\frac{1}{tan\theta} \\ \\ cot\theta=\frac{5}{2\sqrt{14}}*\frac{\sqrt{14}}{\sqrt{14}}=\frac{5\sqrt{14}}{28} \\ \\ cot\theta=\frac{5\sqrt{14}}{28} \end{gathered}[/tex]

please follow directions and refer to rubric Will mark brainliest
screenshots attached thank you I really need this

Answers

Answer:

Step-by-step explanation: To find the x-intercept, substitute in

0

for

y

and solve for

x

. To find the y-intercept, substitute in  0 for x

and solve for  y

x-intercept(s):  735,0

y-intercept(s):   0,490

What ever I'm doing is making the circle equal 410 degrees.

Answers

Step 1

Redraw the cyclic quadrilateral

From the image;

[tex]\begin{gathered} mSTU=220 \\ \text{mSRU}=360\text{ -220=110(sum of angles at a point or in a circle is 3}60^o) \end{gathered}[/tex]

Step 2

Find the measure of angle R

[tex]m\angle R=\frac{1}{2}mSTU[/tex]

[tex]m\angle R=\frac{1}{2}\times220=110[/tex]

Find the measure of angle T

[tex]\begin{gathered} m\angle T=\frac{1}{2}mSRU \\ m\angle T=\frac{1}{2}\times140 \\ m\angle T=70 \end{gathered}[/tex]

Find the measure of angle U

[tex]m\angle U=360-95-110-70=85\text{ ( sum of angles in quadrilateral)}[/tex]

Find the measure of Arc SRU

[tex]\begin{gathered} \text{mSRU}=\text{ }360-\text{mSTU} \\ \text{mSRU}=360-220=140 \end{gathered}[/tex]

Find the measure of Arc mRUT

[tex]\begin{gathered} \text{mRUT}=\text{ 2}\times m\angle S \\ \text{mRUT}=2\times95=190 \end{gathered}[/tex]

Find the measure of mRST

[tex]\begin{gathered} \text{mRST}=2\times m\angle U \\ \text{mRST}=2\times85=170 \end{gathered}[/tex]

How many times greater is the value of 4,563 than the value of 4.563?

Answers

The given number 4,563 is 4558.437 times greater than 4.563.

What is decimal?


Decimals are numbers that have two components, a whole number component and a fractional component, which are separated by a decimal point. 12.5 is a decimal number, for instance.

Using a single notation, decimals can convey both the entire number and the fraction. Whole numbers and fractions are separated from each other in decimals by a point called a decimal point. For instance, in the number 65.4, the entire number is 65, while the fractional half is 4 (10).

The given numbers are 4,563 that is four thousand five hundred sixty- three and 4.563 that is four point five six three.

The difference between these two numbers is 4558.437.

To know more about decimal visit:
https://brainly.com/question/109147

#SPJ9

what is the value of 10-¹

Answers

The value of

[tex]10^{-1}[/tex]

which can be written as:

[tex]\frac{1}{10}[/tex]

is

[tex]0.1[/tex]

Other Questions
water is poured into a right cylindrical tank at a rate of 6 cubic inches per minute. if the radius of the base is 20 incheas long; how fast is the hieght changing when the water level is 12 inches high? Which graph represents the solution to this inequality? a core authentication server is exposed to the internet and is connected to sensitive services. what are some measures you can take to secure the server and prevent it from getting compromised by a hacker? select all that apply. which of the following are referred to as mononuclear phagocytes? check all that apply.question 4 options:a) mast cellsb) eosinophils) basophils) macrophages) dendritic cellsf) monocytesg) lymphocytes Which of the following describes the kingdom of Mali?A.It was a Muslim Empire, ruled by Mansa Musa, and got rich with the gold-salt trade.B.It was an empire in Southern Africa that invented Swahili and controlled trade with Asia.C.It was the first organized kingdom in West Africa and grew rich taxing the Gold-Salt trade.D.It was ruled by Sunni Ali and Askia Muhammad, controlled important trade routes and was conquered by invaders from Morocco. What happens to this enzyme when the pH rises above 11? What is the measure of the base of the rectangle if the area of the triangle is 28ft^2 ?PLSSS ANSWER ASAP genetic change in bacteria can be brought about by genetic change in bacteria can be brought about by transduction. conjugation. mutation. transformation. all of the answers are correct. ufa was successful because it spent lavishly to produce films outside of germany due to the beneficial exchange rate of the german currency after wwi. Match the terms to their correct definitions.1. generation2. rivalry3. continuity4. confidantI competition with another personconsistency, when a change occursgraduallya trusted person with whom youcan share personal informationpeople born in a specific era Divide using synthetic division. Write down the answer as a polynomial.x^3-5x^2-2x+24=0; (x+2) Premises:If I'm a student, then I go to school. If I go to school, then I learn.Conclusion:If I'm a student, then I learn.This is an example of the Law of? The principle of ________ states that organisms succeed one another in a definite, determinable order that can be used to recognize a specific time period. which situation can be represented by this inequality1.25x -6.50 > 50 A. Caleb has a balance of 6.50$ in his savings account and deposits 1.25$ each week. What is X the number of weeks must deposit 1.25$ in order to have a balance of more than 50$ in his savings account?B. Caleb earns 1.25% interest on the balance in his checking account and has to pay a monthly charge of 6.50$. What is X the balance that Caleb must have in his checking account in order to have an ending balance greater than 50$ after interest and fees.C. Caleb charges 1.25$ for gasoline plus 6.50$ per hour for mowing lawns. What is X the number of hours he has to mow lawns to earn more than 50$D. Caleb spends 6.50$ on supplies for a lemonade stand and sells each cup of lemonade for 1.25$. what is X the number of cups of lemonade Caleb must sell to earn profit of more than 50$ if you are on top of mount Everest and do a small hop without moving forward, would you still be pulled down by gravity? explain it and also answer my other newest question. 27 is The same as the product of four and a number how has religion been manipulated to advance conflicts Given that Kelsey has already made 10 pendants how many additional pendants must she make and sell to make a profit of 50 dollars? Determine the number of solutions for the following system of linear equations. If there is only onesolution, find the solution.3x 3y - 3z = 37x 2y - z = -4- 6x 4y - 6z = -1-AnswerKeypadKeyboard ShortcutsSelecting an option will enable input for any required text boxes. If the selected option does not have anyassociated text boxes, then no further input is required.O No SolutionO Only One SolutionX =y =z =O Infinitely Many Solutions A and B are sets of real numbers defined as follows.A = {x|x 2}XB = {x|x < 7}Write A UB and An B using interval notation.If the set is empty, write 0.